Difference between revisions of "2025 USAJMO Problems/Problem 6"
(Created page with "__TOC__ == Problem == Let <math>S</math> be a set of integers with the following properties: <math>\bullet</math> <math>\{ 1, 2, \dots, 2025 \} \subseteq S</math>. <math>\b...") |
(→Solution) |
||
Line 12: | Line 12: | ||
Prove that <math>S</math> contains all positive integers. | Prove that <math>S</math> contains all positive integers. | ||
− | == Solution == | + | == Solution 1== |
− | {{ | + | |
+ | We will show that if <math>\{1, 2, 3, \dots, n\}\in S</math>, then <math>n+1\in S</math>. Note that if <math>n+1</math> is composite, we are done, so we assume that <math>n+1</math> is an odd prime. | ||
+ | |||
+ | Case 1: <math>n+1\ne 2^k\pm 1</math> for some integer <math>k>2</math>. | ||
+ | Since <math>n+2\ne 2^k</math>, we have that <math>2^{v_2 (n+2)}, \frac{n+2}{2^{v_2 (n+2)}}\le n\implies 2^{v_2 (n+2)}, \frac{n+2}{2^{v_2 (n+2)}}\in S\implies n+2\in S</math>. Either <math>v_2(n)=1</math> or <math>v_2(n+2)=1</math>, so <math>2^{v_2(n)+v_2(n+2)}=2^{1+\max(v_2(n), v_2(n+2))}=2\cdot 2^{\max(v_2(n), v_2(n+2))}\le 2\cdot \frac{n+2}{3}\le n\implies 2^{v_2(n)+v_2(n+2)}\in S</math>. Since <math>\gcd(n, n+2)=2</math>, we have that <math>\gcd\left(\frac{n}{2^{v_2(n)}}, \frac{n+2}{2^{v_2(n+2)}}\right)=1</math>, so <cmath>2^{v_2(n)+v_2(n+2)}\cdot \frac{n}{2^{v_2(n)}}\cdot \frac{n+2}{2^{v_2(n+2)}}=n(n+2)\in S\implies n(n+2)+1=(n+1)^2\in S\implies n+1\in S.</cmath> | ||
+ | |||
+ | Case 2: <math>n+1=2^k+1</math> for some integer <math>k>2</math>. | ||
+ | Note that <math>k</math> must be even, otherwise <math>2^k+1\equiv 0\pmod 3</math>. Then we have the following: | ||
+ | <cmath>\left(2^{\frac{k}{2}}+1\right)^2-1=2^{\frac{k}{2}}\left(2^{\frac{k}{2}}+2\right)=2^{\frac{k}{2}+1}\left(2^{\frac{k}{2}-1}+1\right)\in S \implies\left(2^{\frac{k}{2}}+1\right)^2\in S</cmath> | ||
+ | <cmath>\implies 2^{\frac{k}{2}-1}\left(2^{\frac{k}{2}}+1\right)^2 = \left(2^{\frac{k}{2}-1}+1\right)\left(2^k+1\right)-1\in S\implies \left(2^{\frac{k}{2}-1}+1\right)\left(2^k+1\right)\in S\implies 2^k+1\in S,</cmath> | ||
+ | as desired. | ||
+ | |||
+ | Case 3: <math>n+1=2^k-1</math> for some integer <math>k>2</math>. | ||
+ | Let <math>x<k</math> be a positive integer. Then | ||
+ | <cmath>(2^x-1)(2^k-1)-1=2^x\left(2^k-2^{k-x}-1\right)\in S\implies 2^k-1\in S,</cmath> as desired. | ||
+ | |||
+ | Hence, since <math>\{1, 2, 3, \dots, n\}\in S\implies n+1\in S</math>, the set <math>S</math> spans all positive integers. | ||
+ | |||
+ | -mop | ||
==See Also== | ==See Also== | ||
{{USAJMO newbox|year=2025|num-b=5|after=Last Problem}} | {{USAJMO newbox|year=2025|num-b=5|after=Last Problem}} | ||
{{MAA Notice}} | {{MAA Notice}} |
Latest revision as of 12:31, 22 March 2025
Contents
Problem
Let be a set of integers with the following properties:
.
If
and
, then
.
If for some
,
is composite, then all positive divisors of
are in
.
Prove that contains all positive integers.
Solution 1
We will show that if , then
. Note that if
is composite, we are done, so we assume that
is an odd prime.
Case 1: for some integer
.
Since
, we have that
. Either
or
, so
. Since
, we have that
, so
Case 2: for some integer
.
Note that
must be even, otherwise
. Then we have the following:
as desired.
Case 3: for some integer
.
Let
be a positive integer. Then
as desired.
Hence, since , the set
spans all positive integers.
-mop
See Also
2025 USAJMO (Problems • Resources) | ||
Preceded by Problem 5 |
Followed by Last Problem | |
1 • 2 • 3 • 4 • 5 • 6 | ||
All USAJMO Problems and Solutions |
These problems are copyrighted © by the Mathematical Association of America, as part of the American Mathematics Competitions.